Minimum

Algebra Level 3

x x , y y , and z z are real numbers such that x + 2 y + 3 z = 7 x+2y+3z=7 find the minimum of x 2 + y 2 + z 2 x^2+y^2+z^2 .

3.5 3 0 7

This section requires Javascript.
You are seeing this because something didn't load right. We suggest you, (a) try refreshing the page, (b) enabling javascript if it is disabled on your browser and, finally, (c) loading the non-javascript version of this page . We're sorry about the hassle.

3 solutions

Relevant wiki: Lagrange Multipliers

Here is a calculus method,

By Lagrange Multipliers method we set g ( x , y , z ) c = x + 2 y + 3 z 7 g(x,y,z)-c=x+2y+3z-7

f ( x , y , z ) = x 2 + y 2 + z 2 f(x,y,z)=x^2+y^2+z^2 & L ( x , y , z , λ ) = f ( x , y , z ) + λ ( g ( x , y , z ) c ) \mathfrak{L}(x,y,z,\lambda)=f(x,y,z)+\lambda(g(x,y,z)-c)

Now we have Δ x , y , z , λ L ( x , y , z , λ ) = ( L x , L y , L z , L λ ) \Delta_{x,y,z,\lambda}\mathfrak{L}(x,y,z,\lambda) = (\frac{\partial{L}}{\partial{x}},\frac{\partial{L}}{\partial{y}},\frac{\partial{L}}{\partial{z}},\frac{\partial{L}}{\partial{\lambda}})

Δ x , y , z , λ L ( x , y , z , λ ) = ( 2 x + λ , 2 y + 2 λ , 2 z + 3 λ , x + 2 y + 3 z ) \Delta_{x,y,z,\lambda}\mathfrak{L}(x,y,z,\lambda) = (2x+\lambda,2y+2\lambda,2z+3\lambda,x+2y+3z)

Δ x , y , z , λ L ( x , y , z , λ ) = 0 { 2 x + λ = 0 2 y + 2 λ = 0 2 z + 3 λ x + 2 y + 3 z = 7 \Delta_{x,y,z,\lambda}\mathfrak{L}(x,y,z,\lambda) = 0\implies \begin{cases} 2x+\lambda=0 \\ 2y+2\lambda=0 \\ 2z+3\lambda \\ x+2y+3z=7 \end{cases}

Solving we get the critical point ( x , y , z ) = ( 1 2 , 1 , 3 2 ) (x,y,z)=(\frac{1}{2},1,\frac{3}{2})

Moreover This does not assure us it's a point of minimum, but just a critical point. We see f ( 1 2 , 1 , 3 2 ) = 3.5 f(\frac{1}{2},1,\frac{3}{2})=3.5 .

Taking the Hessian Matrix defined by ,

Δ 2 f ( x , y , z ) = ( 2 0 0 0 2 0 0 0 2 ) \begin{aligned} \Delta^2f(x,y,z) = \begin{pmatrix} 2&0&0\\0&2&0\\0&0&2\end{pmatrix}\end{aligned} , we readily observe it's positive definite. So f ( x , y , z ) f(x,y,z) has a local minimum & it's indeed the only stationary point we have. Hence f ( x , y , z ) f ( 1 2 , 1 , 3 2 ) = 3.5 f(x,y,z)\ge f(\frac{1}{2},1,\frac{3}{2}) = 3.5

Long time no see

Peter van der Linden - 4 years, 9 months ago

Using Cauchy-Schwarz inequality, we have:

( 1 2 + 2 2 + 3 2 ) ( x 2 + y 2 + z 2 ) ( x + 2 y + 3 z ) 2 ( 1 + 4 + 9 ) ( x 2 + y 2 + z 2 ) 7 2 x 2 + y 2 + z 2 49 14 = 7 2 = 3.5 \begin{aligned} (1^2+2^2+3^2)(x^2+y^2+z^2) & \ge (x+2y+3z)^2 \\ (1+4+9)(x^2+y^2+z^2) & \ge 7^2 \\ \implies x^2+y^2+z^2 & \ge \frac {49}{14} = \frac 72 = \boxed{3.5} \end{aligned}

Jun Shin
Jan 31, 2016

Apply the Cauchy-Schwarz inequality.
(/(1^2+2^2+3^2/))*(/(x^2+y^2+z^2/))≥/(49/)
/(x^2+y^2+z^2/)≥/(3.5/)
therefore, the minimum is 3.5


You are wrong applying cauchy Schwarz will give

( 1 2 + 2 2 + 3 2 ) ( x 2 + y 2 + z 2 ) ( x + 2 y + 3 z ) 2 \large{\left( { 1 }^{ 2 }+{ 2 }^{ 2 }+{ 3 }^{ 2 } \right) \left( { x }^{ 2 }+{ y }^{ 2 }+{ z }^{ 2 } \right) \ge { \left( x+2y+3z \right) }^{ 2 }}

and I think you should ask for the maximum value of x 2 + y 2 + z 2 x^2+y^2+z^2 because

( x 2 + 2 y 2 + 3 z 2 ) ( x 2 + y 2 + z 2 ) 0 \large{\left( x^{ 2 }+2y^{ 2 }+3z^{ 2 } \right) \ge \left( { x }^{ 2 }+{ y }^{ 2 }+{ z }^{ 2 } \right) \ge { 0 }}

Department 8 - 5 years, 4 months ago

Log in to reply

Sorry, I wrote the /(x+2y+3z/) part wrong. It should be the square of the whole expression. Thanks for pointing it out.

Jun Shin - 5 years, 4 months ago

Log in to reply

No problem it happens

Department 8 - 5 years, 4 months ago

0 pending reports

×

Problem Loading...

Note Loading...

Set Loading...